Aquí tiene una respuesta un poco rebuscada a su pregunta en el afirmativo
Usted observa que $k = \mathrm{gcd}(a,b)^2$ . Después de plodding a través de varios recursos, es porque uno puede Viete Jump:
$$ (a,b) \mapsto \big(a_1, b_1\big) \mapsto \dots \mapsto (k,0) $$
y el gcd es conservado. Una vez que estoy de acuerdo con usted, que $a = \text{gcd}(a,b) \, c$ y $b = \text{gcd}(a,b) \, d$ para que \begin{eqnarray} ab+1 &=& \frac{a^2+b^2}{\mathrm{gcd}(a,b)^2}= c^2 + d^2 \\ &=& \,\text{gcd}(a,b)^2 \, cd + 1 \end{eqnarray}
De este modo, se dan dos condiciones $c, d$ podría resolver (donde los dos $\square$ son diferentes) y $\text{gcd}(c,d)=1$ : \begin{eqnarray*} c^2 - \square \, cd + d^2 &=& 1 \\ c^2 + d^2 &=& \square \end{eqnarray*}
Es de esperar que estas dos ecuaciones lleven a una contradicción.
Añadido el 15/11 La respuesta es definitivamente no . Sea $k = \mathrm{gcd}(a,b)$ Estamos tratando de resolver en números enteros:
\begin{eqnarray} c^2 - k\; cd + d^2 &=& 1 \\ c^2 + d^2 = \square \end{eqnarray}
Como he aprendido, la primera puede resolverse con $(c,d) = (1,0)$ o $(k,1)$ y existe una familia infinita de soluciones utilizando términos consecutivos de una secuencia recursiva [ 2 , 3 ] $$ x_{n+1} = k \, x_n + x_{n-1} $$ A veces hay formas de relacionar los triples pitagóricos con las ecuaciones de Pell [ 1 ] ( Árbol modular de Pitágoras )
$$ x_{n+1}^2 + \frac{1}{2} x_n^2 < \sqrt{x_{n+1}^2 + x_n^2 } = x_{n+1}^2 \sqrt{1 + (x_n/x_{n+1})^2} < x_{n+1}^2 + \frac{1}{2} x_n^2 + 1$$
No puede ser un número entero. Así que cada vez que resolvemos la ecuación de Pell, no podemos también resolver la de Pitágoras. $\quad\quad\square$
Respuesta antigua
Esto se discute en el artículo de Wikipedia sobre Salto de la Vieta :
Ninguno de los seis miembros del comité de problemas australianos pudo resolverlo. Dos de los miembros eran George Szekeres y su esposa, ambos famosos solucionadores y creadores de problemas. [...] El comité del problema lo presentó al jurado de la XXIX OMI marcado con un doble asterisco, que significaba un problema superduro, posiblemente demasiado difícil de plantear. Tras un largo debate, el jurado tuvo finalmente el valor de elegirlo como último problema del concurso. Once estudiantes dieron soluciones perfectas.
Entre los once estaba Bau Chau Ngô (Medalla Fields 2010). Su trabajo sobre el Lemma Fundamental también tiene un sabor saltarín [ 1 , 2 , 3 ] pero está bastante avanzado.
El debate en YouTube también es útil. Estos vídeos ofrecen un debate exhaustivo sobre las diferentes formas de resolver
Puede que no resuelvan directamente su problema, pero proporcionan un contexto histórico e indican posibles estrategias.
En el Wikipedia artículo, el ejemplo de Viete Jumping es OMI 1988/6 -- lo mismo que se pide en la pregunta:
Dejemos que $a,b$ sean enteros positivos tales que $ab+1$ divide $a^2 + b^2$ demostrar que $ \frac{a^2 + b^2}{ab+1}$ es un cuadrado perfecto.
y la solución va en tres pasos
#1 Dejemos que $a, b \geq 0$ ser soluciones a $\frac{a^2 + b^2}{ab+1} = k$ tal que $k$ no es un cuadrado perfecto: $k \neq \square$
#2 A partir de $(a,b)$ podemos intentar generar otra solución $(x,b)$ que resuelve la ecuación cuadrática: $$ x^2 - kb\, x +(b^2 - k) = 0 $$ El mapa $(a,b) \mapsto (a_1,b)$ es nuestro Salto de Vieta Dado que ambos $a, a_1$ son soluciones aceptables que tenemos: $$ (x-a)(x-a_1) = x^2 - (a + a_1) x + aa_1 = 0$$
Por las ecuaciones de Viete (comparando los coeficientes. Descubrimos dos cosas:
#3 Si $a \geq b$ podemos deducir que $a_1 \geq 0$ (es positivo) y además $ a > a_1 \ge 0 $
Resumen Hemos demostrado que dados dos números positivos $a,b$ resolver $\frac{a^2+b^2}{ab+1}=k$ con $k \neq \square$ podemos siempre encontrar otra solución $(a_1,b)$ resolviendo la misma ecuación con $a > a_1$ .
Entonces el salto de Viete consiste en un mapa:
$$ (a,b) \mapsto \left\{\begin{array}{rc} (b,a) & \text{ if } a \leq b \\ (\frac{b^2-k}{a},b) & \text{ if } a \geq b \end{array}\right.$$
Si bien esto hace no resolver su problema para demostrar que $ab+1 \neq \square$ -- sí indica posiblemente por dónde empezar y algunos posibles recursos.
Un uso rápido de la fórmula de Bezout muestra que $ab+1$ también debe dividir $$ \big[(a^2 + b^2) + 2(ab+1)\big] + \big[(a^2 + b^2) - 2(ab+1)\big] = (a+b)^2 +(a-b)^2 $$
y esto podría llevar a su contradicción.
0 votos
Pueden utilizarse estas fórmulas. artofproblemsolving.com/community/
0 votos
Podrían ser útiles, ¡gracias!
0 votos
¿cómo sabes que el valor de $k = \frac{a^2 + b^2}{ab+1}$ es $\text{gcd}(a,b)^2$ ?
0 votos
Aquí hay un enlace a una prueba: google.it/
0 votos
He reducido tu problema para encontrar una solución sobre los enteros a un par de formas cuadráticas. Este es probablemente un problema muy difícil. Muy poca información que pude obtener en Google para este tema.
0 votos
@cactus314 de $m$ y $n$ exactamente uno es impar, y el que es par es divisible por $8$ . Usa eso $(n^2+m^2+1)/2$ es impar (utilice ese impar $^2-1=0\mod4$ ). La secuencia recursiva $M_0=0,M_1=1,M_{k}=2M_{k-1}+M_{k-2}$ , produce $0,1,2,5,12,29,70,169,408,985,2378,5741,13860$ etc, los valores consecutivos no nulos dan $n,m$ para la cual la fracción $\displaystyle\frac{(n^2+m^2+1)(n^2+m^2-1)}{2mn(n+m)(n-m)} = \pm2$ . Lo he buscado en Google: oeis.org/A000129 Números Pell, a veces también llamados números lambda.
0 votos
Hay un problema relacionado que he enumerado aquí: math.stackexchange.com/questions/1998292/ que es gracias a la observación de @Mirko sobre la relación entre los números Pell y nuestro. La afirmación allí es más fuerte que la afirmación aquí, ya que implica que lo anterior es cierto
0 votos
Arriba quería decir que "el que es par es divisible por 8" proporcionado que $k$ es impar, donde suponemos que la relación $\displaystyle\frac{(n^2+m^2)^21}{2mn(n^2m^2)}=k$ es un número entero. Una posible sustitución a considerar: Sea $n^2=x$ , $m^2=y$ , $nm=z$ , entonces obtenemos $(xkz)^2+(y+kz)^2=2z^2(k^21)+1$ el grado en esta ecuación es menor, pero tiene una variable extra, tiene $x,y,z$ en lugar de sólo $n,m$ , teniendo en cuenta $k$ un parámetro. Si $k=2$ entonces las únicas soluciones son los números de Pell, ya que entonces $n=±\sqrt{2m^2±1}+m$ Por ejemplo $m=2,n=\sqrt{2m^2+1}+m=5$ o $m=5,n=\sqrt{2m^2-1}+m=12$ . OP sugiere $k=l^2$ .
0 votos
También es posible demostrar que las únicas soluciones enteras $n>m > 0$ a $\frac{(n^2 + m^2)^2 - 1}{2nm(n^2 - m^2)} = 2$ son números Pell consecutivos observando que si $(n,m)$ es una solución a la ecuación anterior, entonces también lo son $(n + 2m,m)$ y $(m,n-2m)$ .
0 votos
@Michael Hoefnagel : Siento las molestias con mi post, lo he borrado. Sólo voy a deshacerlo, si he encontrado la solución correcta por completo.
0 votos
No te preocupes, espero que consigas resolver el problema.